6
$\begingroup$

Let $E $ be a (possibly nonconnective) spectrum. Suppose $E \wedge K = 0$ (where $K$ is complex $K$-theory). Does it follow that $E = 0$?

$\endgroup$

2 Answers 2

8
$\begingroup$

Sure. Smashing a based space with a spectrum is equivalent to smashing its suspension spectrum with that spectrum. So it suffices to give a nontrivial space whose reduced $K$-homology is trivial. A classical example due to Luke Hodgkin is $Coker J$. See Hodgkin, Luke; Snaith, Victor. The K-theory of some more well-known spaces. Illinois J. Math. 22 (1978), no. 2, 270–278.

$\endgroup$
3
  • 2
    $\begingroup$ Peter, you don't mean "sure". You mean "surely not". $\endgroup$ Jun 10, 2012 at 1:56
  • $\begingroup$ Right you are Tom, I surely mean surely not. But Akhil understood. $\endgroup$
    – Peter May
    Jun 10, 2012 at 2:07
  • 3
    $\begingroup$ Tom, actually the original Sure'' was in answer to the original question. And for that question the answer Sure'' is clearly right. $\endgroup$
    – Peter May
    Jun 10, 2012 at 2:11
10
$\begingroup$

Another type of example: Let $E$ be the mod $p$ homology spectrum. The integral homology groups of the periodic $K$-theory spectrum are rational vector spaces, i.e. the integral homology groups of the mod $p$ spectrum are zero.

$\endgroup$
3
  • 8
    $\begingroup$ Moreover, you can prove this without too much computation. Both $K$ and $H/p$ are complex oriented, so a quite conceptual argument shows that the additive formal group law (associated to $H/p$) and the multiplicative one (associated to $K$) become isomorphic over $\pi_*(K\wedge H/p)$. A simple algebraic argument now shows that $\pi_*(K\wedge H/p)=0$. $\endgroup$ Jun 10, 2012 at 11:36
  • 2
    $\begingroup$ Neil, that's nice. The proof that I know consists of examining the effect of the Bott map $S^2\wedge BU\to BU$ on integral homology. This comes down to calculating the Chern class(es) of the generator of $\tilde K(S^{2n})$, which can be done using the Chern character. $\endgroup$ Jun 10, 2012 at 15:01
  • 1
    $\begingroup$ @Neil: I know that argument works more generally. Would you mind posting it as a separate answer? $\endgroup$ Jun 10, 2012 at 15:32

Your Answer

By clicking “Post Your Answer”, you agree to our terms of service and acknowledge you have read our privacy policy.

Not the answer you're looking for? Browse other questions tagged or ask your own question.